IASbaba Prelims 60 Days Plan, Rapid Revision Series (RaRe)
Archives
Hello Friends
The 60 Days Rapid Revision (RaRe) Series is IASbaba’s Flagship Initiative recommended by Toppers and loved by the aspirants’ community every year.
It is the most comprehensive program which will help you complete the syllabus, revise and practice tests on a daily basis. The Programme on a daily basis includes
Daily Prelims MCQs from Static (Monday – Saturday)
- Daily Static Quiz will cover all the topics of static subjects – Polity, History, Geography, Economics, Environment and Science and technology.
- 20 questions will be posted daily and these questions are framed from the topics mentioned in the schedule.
- It will ensure timely and streamlined revision of your static subjects.
Daily Current Affairs MCQs (Monday – Saturday)
- Daily 5 Current Affairs questions, based on sources like ‘The Hindu’, ‘Indian Express’ and ‘PIB’, would be published from Monday to Saturday according to the schedule.
Daily CSAT Quiz (Monday – Friday)
- CSAT has been an Achilles heel for many aspirants.
- Daily 5 CSAT Questions will be published.
Note – Daily Test of 20 static questions, 10 current affairs, and 5 CSAT questions. (35 Prelims Questions) in QUIZ FORMAT will be updated on a daily basis.
To Know More about 60 Days Rapid Revision (RaRe) Series – CLICK HERE
60 Day Rapid Revision (RaRe) Series Schedule – CLICK HERE
Important Note
- Comment your Scores in the Comment Section. This will keep you accountable, responsible and sincere in days to come.
- It will help us come out with the Cut-Off on a Daily Basis.
- Let us know if you enjoyed today’s test 🙂
- You can post your comments in the given format
- (1) Your Score
- (2) Matrix Meter
- (3) New Learning from the Test
Test-summary
0 of 35 questions completed
Questions:
- 1
- 2
- 3
- 4
- 5
- 6
- 7
- 8
- 9
- 10
- 11
- 12
- 13
- 14
- 15
- 16
- 17
- 18
- 19
- 20
- 21
- 22
- 23
- 24
- 25
- 26
- 27
- 28
- 29
- 30
- 31
- 32
- 33
- 34
- 35
Information
The following Test is based on the syllabus of 60 Days Plan-2023 for UPSC IAS Prelims 2022.
To view Solutions, follow these instructions:
- Click on – ‘Start Test’ button
- Solve Questions
- Click on ‘Test Summary’ button
- Click on ‘Finish Test’ button
- Now click on ‘View Questions’ button – here you will see solutions and links.
You have already completed the test before. Hence you can not start it again.
Test is loading...
You must sign in or sign up to start the test.
You have to finish following test, to start this test:
Results
0 of 35 questions answered correctly
Your time:
Time has elapsed
You have scored 0 points out of 0 points, (0)
Average score |
|
Your score |
|
Categories
- Not categorized 0%
Pos. | Name | Entered on | Points | Result |
---|---|---|---|---|
Table is loading | ||||
No data available | ||||
- 1
- 2
- 3
- 4
- 5
- 6
- 7
- 8
- 9
- 10
- 11
- 12
- 13
- 14
- 15
- 16
- 17
- 18
- 19
- 20
- 21
- 22
- 23
- 24
- 25
- 26
- 27
- 28
- 29
- 30
- 31
- 32
- 33
- 34
- 35
- Answered
- Review
-
Question 1 of 35
1. Question
With respect to the SAVE (Saving Asia’s Vultures from Extinction) project, consider the following statements:
- The safe zones established for the captive-bred birds to be released under SAVE project lies completely in Indian territory.
- Bombay Natural History is a partner for SAVE project.
- SAVE is currently engaged in advocacy to more effectively remove diclofenac from veterinary use.
- SAVE meets once in every three years to report on the activities of the partners and to review regional priorities.
How many of the above statements are correct?
Correct
Solution (b)
Statement 1 Statement 2 Statement 3 Statement 4 Incorrect Correct Correct Incorrect The safe zones established for the captive-bred birds to be released under SAVE project have a radius of 100km and the consortium has identified six such areas – some of which cross national borders into Nepal, Pakistan and Bangladesh. SAVE partners: 1. Bombay Natural History Society
2. Bird Conservation Nepal, RSPB (UK)
3. National Trust for Nature Conservation (Nepal)
4. International Centre for Birds of Prey (UK) and
5. Zoological Society of London.
SAVE is currently engaged in advocacy to more effectively remove diclofenac from veterinary use, creation of Vulture Safe Zones and formulation of breeding Centres and release programmes. SAVE meets once a year to report on the activities of the partners and to review regional priorities in the form of an annually updated Blueprint Recovery Plan, with national breakdowns for key actions. Note:
SAVE (Saving Asia’s Vultures from Extinction):
- The consortium of like-minded, regional and international organizations, created to oversee and co-ordinate conservation, campaigning and fundraising activities to help the plight of south Asia’s vultures.
- In order to ensure that the species affected do not disappear completely from the wild, SAVE identified a number of priorities viz.
- To establish a number of vast ‘safe zones’ for the captive-bred birds to be released within. The areas have a radius of 100km and the consortium has identified six such areas – some of which cross national borders into Nepal, Pakistan and Bangladesh.
- Establishment of a captive-breeding programme that would provide the birds to be released back into the wider environment, once it was safe to do so.
- SAVE meets once a year to report on the activities of the partners and to review regional priorities in the form of an annually updated Blueprint Recovery Plan, with national breakdowns for key actions.
- There are also two main expert committees: the Technical Advisory Committee (TAC) and the Fundraising, Advocacy, and Communications Committee (FACC).
- Vultures have been added to the list of ‘critically endangered’ species by the International Union for Conservation of Nature (IUCN).
Objective:
- To save three critically important species of vultures from extinction through a single programme.
- Oriental White-backed(Gyps bengalensis)
- Long-billed (Gyps indicus)
- Slender-billed vulture (Gyps tenuirostris)
- SAVE is currently engaged in advocacy to more effectively remove diclofenac from veterinary use, creation of Vulture Safe Zones and formulation of breeding Centres and release programmes.
SAVE partners:
- Bombay Natural History Society
- Bird Conservation Nepal, RSPB (UK)
- National Trust for Nature Conservation (Nepal)
- International Centre for Birds of Prey (UK) and
- Zoological Society of London.
Incorrect
Solution (b)
Statement 1 Statement 2 Statement 3 Statement 4 Incorrect Correct Correct Incorrect The safe zones established for the captive-bred birds to be released under SAVE project have a radius of 100km and the consortium has identified six such areas – some of which cross national borders into Nepal, Pakistan and Bangladesh. SAVE partners: 1. Bombay Natural History Society
2. Bird Conservation Nepal, RSPB (UK)
3. National Trust for Nature Conservation (Nepal)
4. International Centre for Birds of Prey (UK) and
5. Zoological Society of London.
SAVE is currently engaged in advocacy to more effectively remove diclofenac from veterinary use, creation of Vulture Safe Zones and formulation of breeding Centres and release programmes. SAVE meets once a year to report on the activities of the partners and to review regional priorities in the form of an annually updated Blueprint Recovery Plan, with national breakdowns for key actions. Note:
SAVE (Saving Asia’s Vultures from Extinction):
- The consortium of like-minded, regional and international organizations, created to oversee and co-ordinate conservation, campaigning and fundraising activities to help the plight of south Asia’s vultures.
- In order to ensure that the species affected do not disappear completely from the wild, SAVE identified a number of priorities viz.
- To establish a number of vast ‘safe zones’ for the captive-bred birds to be released within. The areas have a radius of 100km and the consortium has identified six such areas – some of which cross national borders into Nepal, Pakistan and Bangladesh.
- Establishment of a captive-breeding programme that would provide the birds to be released back into the wider environment, once it was safe to do so.
- SAVE meets once a year to report on the activities of the partners and to review regional priorities in the form of an annually updated Blueprint Recovery Plan, with national breakdowns for key actions.
- There are also two main expert committees: the Technical Advisory Committee (TAC) and the Fundraising, Advocacy, and Communications Committee (FACC).
- Vultures have been added to the list of ‘critically endangered’ species by the International Union for Conservation of Nature (IUCN).
Objective:
- To save three critically important species of vultures from extinction through a single programme.
- Oriental White-backed(Gyps bengalensis)
- Long-billed (Gyps indicus)
- Slender-billed vulture (Gyps tenuirostris)
- SAVE is currently engaged in advocacy to more effectively remove diclofenac from veterinary use, creation of Vulture Safe Zones and formulation of breeding Centres and release programmes.
SAVE partners:
- Bombay Natural History Society
- Bird Conservation Nepal, RSPB (UK)
- National Trust for Nature Conservation (Nepal)
- International Centre for Birds of Prey (UK) and
- Zoological Society of London.
-
Question 2 of 35
2. Question
With respect to the Prevention of Cruelty to Animals Act, 1960, consider the following statements:
- Though the Act provides for punishment for causing unnecessary cruelty and suffering to animals, it doesn’t discuss the different forms of cruelty, exceptions, and killing of a suffering animal.
- The Act enshrines the provisions relating to the exhibition of the performing animals, and offenses committed against the performing animals.
- A prosecution for an offence against this Act shall not be instituted after the expiration of Six months from the date of the commission of the offence.
How many of the above statements are correct?
Correct
Solution (a)
Statement 1 Statement 2 Statement 3 Incorrect Correct Incorrect Prevention of Cruelty to Animals Act, 1960 provides for punishment for causing unnecessary cruelty and suffering to animals. The Act defines animals and different forms of animals.
Discusses different forms of cruelty, exceptions, and killing of a suffering animal in case any cruelty has been committed against it, so as to relieve it from further suffering.
The Act enshrines the provisions relating to the exhibition of the performing animals, and offenses committed against the performing animals. The Act provides that a prosecution for an offence against this Act shall not be instituted after the expiration of three months from the date of the commission of the offence. Note:
Prevention of Cruelty to Animals Act, 1960:
- The legislative intent of the Act is to ‘prevent the infliction of unnecessary pain or suffering on animals’.
- The Animal Welfare Board of India (AWBI) was established in 1962 under Section 4 of the Act.
- This Act provides for punishment for causing unnecessary cruelty and suffering to animals. The Act defines animals and different forms of animals.
- Discusses different forms of cruelty, exceptions, and killing of a suffering animal in case any cruelty has been committed against it, so as to relieve it from further suffering.
- Provides the guidelines relating to experimentation on animals for scientific purposes.
- The Act enshrines the provisions relating to the exhibition of the performing animals, and offenses committed against the performing animals.
- This Act provides for the limitation period of 3 months beyond which no prosecution shall lie for any offenses under this Act.
Incorrect
Solution (a)
Statement 1 Statement 2 Statement 3 Incorrect Correct Incorrect Prevention of Cruelty to Animals Act, 1960 provides for punishment for causing unnecessary cruelty and suffering to animals. The Act defines animals and different forms of animals.
Discusses different forms of cruelty, exceptions, and killing of a suffering animal in case any cruelty has been committed against it, so as to relieve it from further suffering.
The Act enshrines the provisions relating to the exhibition of the performing animals, and offenses committed against the performing animals. The Act provides that a prosecution for an offence against this Act shall not be instituted after the expiration of three months from the date of the commission of the offence. Note:
Prevention of Cruelty to Animals Act, 1960:
- The legislative intent of the Act is to ‘prevent the infliction of unnecessary pain or suffering on animals’.
- The Animal Welfare Board of India (AWBI) was established in 1962 under Section 4 of the Act.
- This Act provides for punishment for causing unnecessary cruelty and suffering to animals. The Act defines animals and different forms of animals.
- Discusses different forms of cruelty, exceptions, and killing of a suffering animal in case any cruelty has been committed against it, so as to relieve it from further suffering.
- Provides the guidelines relating to experimentation on animals for scientific purposes.
- The Act enshrines the provisions relating to the exhibition of the performing animals, and offenses committed against the performing animals.
- This Act provides for the limitation period of 3 months beyond which no prosecution shall lie for any offenses under this Act.
-
Question 3 of 35
3. Question
With respect to the Indian Forest Act, 1927, consider the following statements:
- It granted legal recognition to the rights of traditional forest dwelling communities.
- It recognised the land rights and usage rights but denied the rights to protect and conserve.
- It categorizes forests into Reserve Forest, Protected Forest and Village Forest.
How many of the above statements are correct?
Correct
Solution (b)
Statement 1 Statement 2 Statement 3 Correct Incorrect Correct This Act recognised forest dwellers’ rights and makes conservation more accountable. The Act basically does two things:
1. Grants legal recognition to the rights of traditional forest dwelling communities, partially correcting the injustice caused by the forest laws, and
2. Makes a beginning towards giving communities and the public a voice in forest and wildlife conservation.
The law recognised three types of rights: 1. Land Rights
2. Use Rights
3. Right to Protect and Conserve
It categorizes forests into Reserve Forest, Protected Forest and Village Forest. Indian Forest Act, 1927:
- This Act recognised forest dwellers’ rights and makes conservation more accountable.
- The Act basically does two things:
- Grants legal recognition to the rights of traditional forest dwelling communities, partially correcting the injustice caused by the forest laws, and
- Makes a beginning towards giving communities and the public a voice in forest and wildlife conservation.
The law recognised three types of rights:
- Land Rights: Land rights are given to people, who have been cultivating land prior to December, 13, 2005.
- Use Rights: The law provides for rights to use and/or collect the minor forest produce things like tendu patta, herbs, medicinal plants etc ‘that has been traditionally collected, use of grazing grounds and water bodies and use of traditional areas by nomadic or pastoralist communities i.e., communities that move with their herds, as opposed to practicing settled agriculture.
- Right to Protect and Conserve: The law gives rights to protect and manage the forests to people of village communities.
The Act also categorises forests into three categories:
- Reserve forest:
- These forests are the most restricted forests and may be constituted by the State Government on any forest land or waste land which is the property of the Government or on which the Government has proprietary rights.
- In reserved forests, most uses by local people are prohibited, unless specifically allowed by a Forest Officer in the course of settlement.
- Protected forest:
- The State Government is empowered to constitute any land other than reserved forests as protected forests over which the Government has proprietary rights.
- Under ‘Protected Forests’, the Government retains the power to issue rules regarding the use of such forests and retains the power to reserve the specific tree species in the protected forests.
- This power has been used to establish State control over trees, whose timber, fruit or other non-wood products have revenue-raising potential.
- Village forest:
- ‘Village forests’ are the one in which the State Government may assign to ‘any village community the rights of Government to or over any land which has been constituted a reserved forest’.
Incorrect
Solution (b)
Statement 1 Statement 2 Statement 3 Correct Incorrect Correct This Act recognised forest dwellers’ rights and makes conservation more accountable. The Act basically does two things:
1. Grants legal recognition to the rights of traditional forest dwelling communities, partially correcting the injustice caused by the forest laws, and
2. Makes a beginning towards giving communities and the public a voice in forest and wildlife conservation.
The law recognised three types of rights: 1. Land Rights
2. Use Rights
3. Right to Protect and Conserve
It categorizes forests into Reserve Forest, Protected Forest and Village Forest. Indian Forest Act, 1927:
- This Act recognised forest dwellers’ rights and makes conservation more accountable.
- The Act basically does two things:
- Grants legal recognition to the rights of traditional forest dwelling communities, partially correcting the injustice caused by the forest laws, and
- Makes a beginning towards giving communities and the public a voice in forest and wildlife conservation.
The law recognised three types of rights:
- Land Rights: Land rights are given to people, who have been cultivating land prior to December, 13, 2005.
- Use Rights: The law provides for rights to use and/or collect the minor forest produce things like tendu patta, herbs, medicinal plants etc ‘that has been traditionally collected, use of grazing grounds and water bodies and use of traditional areas by nomadic or pastoralist communities i.e., communities that move with their herds, as opposed to practicing settled agriculture.
- Right to Protect and Conserve: The law gives rights to protect and manage the forests to people of village communities.
The Act also categorises forests into three categories:
- Reserve forest:
- These forests are the most restricted forests and may be constituted by the State Government on any forest land or waste land which is the property of the Government or on which the Government has proprietary rights.
- In reserved forests, most uses by local people are prohibited, unless specifically allowed by a Forest Officer in the course of settlement.
- Protected forest:
- The State Government is empowered to constitute any land other than reserved forests as protected forests over which the Government has proprietary rights.
- Under ‘Protected Forests’, the Government retains the power to issue rules regarding the use of such forests and retains the power to reserve the specific tree species in the protected forests.
- This power has been used to establish State control over trees, whose timber, fruit or other non-wood products have revenue-raising potential.
- Village forest:
- ‘Village forests’ are the one in which the State Government may assign to ‘any village community the rights of Government to or over any land which has been constituted a reserved forest’.
-
Question 4 of 35
4. Question
Consider the following statements regarding Wildlife Protection Act, 1972:
- Animals listed in schedule 1 and parts II of schedule 2 have absolute protection.
- Animals listed in schedule 4 are called ‘vermin’ which can be hunted.
Choose the correct code:
Correct
Solution (a)
Statement 1 Statement 2 Correct Incorrect Animals listed in schedule 1 and parts II of schedule 2 have absolute protection. Animals listed in schedule 5 are called ‘vermin’ which can be hunted. Note:
Wildlife Protection Act, 1972:
Objectives:
- To safeguard and develop the environment.
- To carry out the conclusions reached at the 1972 United Nations Conference on the Human Environment in Stockholm.
- To punish anyone who harms the environment.
- Enforcing environmental regulations in locations not covered by existing legislation.
- Give the central government complete authority to enact severe environmental protection measures.
- The main purpose of the law is to ensure the protection of wildlife, birds, and plants.
- The law gives the federal government the power to declare certain areas such as a sanctuary or a national park, wildlife hunting bans and bring punishment for their violation.
Schedules:
- Schedule 1 – Protection to Endangered species, from hunting and poaching, unless the animal possess threat to Human lives. Strict penalties and punishment to offenders.
- Schedule 2 – Protection of animals from hunting and poaching, for the purposes of trading.
- Schedule 3 – Protection of animals not considered endangered, in this schedule penalties and punishments are not as harsh as Schedule 1 & 2.
- Schedule 4 – Same as Schedule 3.
- Schedule 5 – Vermins (common crows, fruit bats, rats, and mice) that can be hunted, as they are carriers of diseases. Section 62, of the Act, gives government power to declare an animal a Vermin. It is done when an animal is considered dangerous for humans or crops.
- Schedule 6 – Restricted plants that require specific permission for cultivating.
Incorrect
Solution (a)
Statement 1 Statement 2 Correct Incorrect Animals listed in schedule 1 and parts II of schedule 2 have absolute protection. Animals listed in schedule 5 are called ‘vermin’ which can be hunted. Note:
Wildlife Protection Act, 1972:
Objectives:
- To safeguard and develop the environment.
- To carry out the conclusions reached at the 1972 United Nations Conference on the Human Environment in Stockholm.
- To punish anyone who harms the environment.
- Enforcing environmental regulations in locations not covered by existing legislation.
- Give the central government complete authority to enact severe environmental protection measures.
- The main purpose of the law is to ensure the protection of wildlife, birds, and plants.
- The law gives the federal government the power to declare certain areas such as a sanctuary or a national park, wildlife hunting bans and bring punishment for their violation.
Schedules:
- Schedule 1 – Protection to Endangered species, from hunting and poaching, unless the animal possess threat to Human lives. Strict penalties and punishment to offenders.
- Schedule 2 – Protection of animals from hunting and poaching, for the purposes of trading.
- Schedule 3 – Protection of animals not considered endangered, in this schedule penalties and punishments are not as harsh as Schedule 1 & 2.
- Schedule 4 – Same as Schedule 3.
- Schedule 5 – Vermins (common crows, fruit bats, rats, and mice) that can be hunted, as they are carriers of diseases. Section 62, of the Act, gives government power to declare an animal a Vermin. It is done when an animal is considered dangerous for humans or crops.
- Schedule 6 – Restricted plants that require specific permission for cultivating.
-
Question 5 of 35
5. Question
Which of the following are the powers of the Central Government under the Environment (Protection) Act, 1986?
- It can set the standards for the emissions of pollutants from any source.
- It can lay down the procedures and safeguards for the handling of hazardous substances.
- It can compel a state government to take prior approval for directing any forest land for non-forest purposes.
How many of the above statements are correct?
Correct
Solution (b)
Statement 1 Statement 2 Statement 3 Correct Correct Incorrect It can set the standards for the emissions of pollutants from any source. It can lay down the procedures and safeguards for the handling of hazardous substances. The Forest (Conservation) Act, 1980(Not EPA 1986)compel a state government to take prior approval for directing any forest land for non-forest purposes. Notes:
Under the Environment (Protection) Act, 1986, the power of the Central Government includes:
- Planning and execution of programmes to prevent, control and abate environmental pollution on an all-India basis.
- Making rules in respect of all matters pertaining to environmental protection as well as to pollution.
- Setting standards for the emission of pollutants from any source.
- Laying down the standards for environmental quality.
- Laying down procedures and safeguards for the handling of hazardous.
- Assessing manufacturing processes, materials and substances which could cause environmental pollution.
- Restricting areas in which industries or other establishments are to be set up.
Incorrect
Solution (b)
Statement 1 Statement 2 Statement 3 Correct Correct Incorrect It can set the standards for the emissions of pollutants from any source. It can lay down the procedures and safeguards for the handling of hazardous substances. The Forest (Conservation) Act, 1980(Not EPA 1986)compel a state government to take prior approval for directing any forest land for non-forest purposes. Notes:
Under the Environment (Protection) Act, 1986, the power of the Central Government includes:
- Planning and execution of programmes to prevent, control and abate environmental pollution on an all-India basis.
- Making rules in respect of all matters pertaining to environmental protection as well as to pollution.
- Setting standards for the emission of pollutants from any source.
- Laying down the standards for environmental quality.
- Laying down procedures and safeguards for the handling of hazardous.
- Assessing manufacturing processes, materials and substances which could cause environmental pollution.
- Restricting areas in which industries or other establishments are to be set up.
-
Question 6 of 35
6. Question
Which of the following are the objectives of National Forest Policy, 1988?
- Protect, improve, and enhance the production of minor forest produce with regard to the generation of employment for the tribals.
- Checking the extension of sand-dunes in the desert areas of Rajasthan and along the coastal tracts.
Choose the correct code:
Correct
Solution (c)
Statement 1 Statement 2 Correct Correct Protect, improve, and enhance the production of minor forest produce with regard to the generation of employment for the tribals. Checking the extension of sand-dunes in the desert areas of Rajasthan and along the coastal tracts. Notes:
National Forest Policy, 1988- The basic objectives that should govern the National Forest Policy are:
- Maintenance of environmental stability through preservation and, where necessary, restoration of the ecological balance that has been adversely disturbed by serious depletion of the forests of the country.
- Conserving the natural heritage of the country by preserving the remaining natural forests with the vast variety of flora and fauna, which represent the remarkable biological diversity and genetic resources of the country.
- Meeting the requirements of fuel-wood, fodder, minor forest produce and small timber of the rural and tribal populations.
- Increasing the productivity of forests to meet essential national needs. And thus increasing tribal employment.
- Encouraging efficient utilisation of forest produce and maximising substitution of wood.
- Checking soil erosion and denudation in the catchment areas of rivers, lakes, reservoirs in the interest of soil and water conservation, for mitigating floods and droughts and for the retardation of siltation of reservoirs. Checking the extension of sand-dunes in the desert areas of Rajasthan and along the coastal tracts.
- Increasing substantially the forest/tree cover in the country through massive afforestation and social forestry programmes, especially on all denuded, degraded and unproductive lands.
- Creating a massive people’s movement with the involvement of women, for achieving these objectives and to minimise pressure on existing forests.
Incorrect
Solution (c)
Statement 1 Statement 2 Correct Correct Protect, improve, and enhance the production of minor forest produce with regard to the generation of employment for the tribals. Checking the extension of sand-dunes in the desert areas of Rajasthan and along the coastal tracts. Notes:
National Forest Policy, 1988- The basic objectives that should govern the National Forest Policy are:
- Maintenance of environmental stability through preservation and, where necessary, restoration of the ecological balance that has been adversely disturbed by serious depletion of the forests of the country.
- Conserving the natural heritage of the country by preserving the remaining natural forests with the vast variety of flora and fauna, which represent the remarkable biological diversity and genetic resources of the country.
- Meeting the requirements of fuel-wood, fodder, minor forest produce and small timber of the rural and tribal populations.
- Increasing the productivity of forests to meet essential national needs. And thus increasing tribal employment.
- Encouraging efficient utilisation of forest produce and maximising substitution of wood.
- Checking soil erosion and denudation in the catchment areas of rivers, lakes, reservoirs in the interest of soil and water conservation, for mitigating floods and droughts and for the retardation of siltation of reservoirs. Checking the extension of sand-dunes in the desert areas of Rajasthan and along the coastal tracts.
- Increasing substantially the forest/tree cover in the country through massive afforestation and social forestry programmes, especially on all denuded, degraded and unproductive lands.
- Creating a massive people’s movement with the involvement of women, for achieving these objectives and to minimise pressure on existing forests.
-
Question 7 of 35
7. Question
The National Green Tribunal is competent to hear matters related to which of the following laws?
- The Wildlife (Protection) Act, 1972
- The Public Liability Insurance Act, 1991
- The Biological Diversity Act, 2002
- Forest Rights Act, 2006
Select the correct answer using the code given below:
Correct
Solution (b)
Statement 1 Statement 2 Statement 3 Statement 4 Incorrect Correct Correct Incorrect The Wildlife (Protection) Act, 1972: Not listed in Schedule 1 of the NGT Act, 2010. Therefore, NGT does not hear matters related to this act.
The Public Liability Insurance Act, 1991: Listed in Schedule 1 of the NGT Act, 2010. Therefore, NGT does hear matters related to this act.
The Biological Diversity Act, 2002: Listed in Schedule 1 of the NGT Act, 2010. Therefore, NGT does hear matters related to this act.
Forest Rights Act, 2006: Not listed in Schedule 1 of the NGT Act, 2010. Therefore, NGT does not hear matters related to this act.
Note:
National Green Tribunal:
- It is a specialised body set up under the National Green Tribunal Act (2010) for effective and expeditious disposal of cases relating to environmental protection and conservation of forests and other natural resources.
- With the establishment of the NGT, India became the third country in the world to set up a specialised environmental tribunal, only after Australia and New Zealand, and the first developing country to do so.
- NGT is mandated to make disposal of applications or appeals finally within 6 months of filing of the same.
- The NGT has five places of sittings, New Delhi is the Principal place of sitting and Bhopal, Pune, Kolkata and Chennai are the other four.
The NGT deals with civil cases under the seven laws related to the environment, which are listed in Schedule 1 of the NGT Act,2010. These include:
- The Water (Prevention and Control of Pollution) Act, 1974,
- The Water (Prevention and Control of Pollution) Cess Act, 1977,
- The Forest (Conservation) Act, 1980,
- The Air (Prevention and Control of Pollution) Act, 1981,
- The Environment (Protection) Act, 1986,
- The Public Liability Insurance Act, 1991 and
- The Biological Diversity Act, 2002.
Any violation pertaining to these laws or any decision taken by the Government under these laws can be challenged before the NGT.
Incorrect
Solution (b)
Statement 1 Statement 2 Statement 3 Statement 4 Incorrect Correct Correct Incorrect The Wildlife (Protection) Act, 1972: Not listed in Schedule 1 of the NGT Act, 2010. Therefore, NGT does not hear matters related to this act.
The Public Liability Insurance Act, 1991: Listed in Schedule 1 of the NGT Act, 2010. Therefore, NGT does hear matters related to this act.
The Biological Diversity Act, 2002: Listed in Schedule 1 of the NGT Act, 2010. Therefore, NGT does hear matters related to this act.
Forest Rights Act, 2006: Not listed in Schedule 1 of the NGT Act, 2010. Therefore, NGT does not hear matters related to this act.
Note:
National Green Tribunal:
- It is a specialised body set up under the National Green Tribunal Act (2010) for effective and expeditious disposal of cases relating to environmental protection and conservation of forests and other natural resources.
- With the establishment of the NGT, India became the third country in the world to set up a specialised environmental tribunal, only after Australia and New Zealand, and the first developing country to do so.
- NGT is mandated to make disposal of applications or appeals finally within 6 months of filing of the same.
- The NGT has five places of sittings, New Delhi is the Principal place of sitting and Bhopal, Pune, Kolkata and Chennai are the other four.
The NGT deals with civil cases under the seven laws related to the environment, which are listed in Schedule 1 of the NGT Act,2010. These include:
- The Water (Prevention and Control of Pollution) Act, 1974,
- The Water (Prevention and Control of Pollution) Cess Act, 1977,
- The Forest (Conservation) Act, 1980,
- The Air (Prevention and Control of Pollution) Act, 1981,
- The Environment (Protection) Act, 1986,
- The Public Liability Insurance Act, 1991 and
- The Biological Diversity Act, 2002.
Any violation pertaining to these laws or any decision taken by the Government under these laws can be challenged before the NGT.
-
Question 8 of 35
8. Question
Which of the following are established under the provisions of the Environment (Protection) Act, 1986?
- Environment Pollution (Prevention and Control) Authority
- National Tiger Conservation Authority
- Central Ground Water Authority
- National Ganga Council
- Central Pollution Control Board
Select the correct answer using the code given below:
Correct
Solution (b)
Statement 1 Statement 2 Statement 3 Statement 4 Statement 5 Correct Incorrect Correct Correct Incorrect Environment Pollution (Prevention and Control) Authority: It is established under the provisions of the Environment (Protection) Act, 1986.
National Tiger Conservation Authority (NTCA): It is a statutory body constituted under provisions of the Wildlife (Protection) Act, 1972.
Central Ground Water Authority: It is established under the provisions of the Environment (Protection) Act, 1986.
National Ganga Council: It is established under the provisions of the Environment (Protection) Act, 1986.
The Central Pollution Control Board (CPCB): It is a statutory organisation constituted under the Water (Prevention and Control of Pollution) Act, 1974.
Note:
Environment (Protection) Act, 1986:
The Environment (Protection) Act was enacted in the year 1986.
It was enacted with the main objective to provide the protection and improvement of the environment and for matters connected therewith.
Aims and Objectives of the Environment Protection Act, 1986:
- Implementing the decisions made at the United Nations Conference on Human Environment held in Stockholm.
- Creation of a government authority to regulate industry that can issue direct orders including closure orders.
- Coordinating activities of different agencies that are operating under the existing laws.
- Enacting regular laws for the protection of the environment.
- Imposing punishments and penalties on those who endanger the environment, safety and health. For each failure or contravention, the punishment includes a prison term of up to five years or a fine of up to Rs. 1 lakh, or both. This can also be extended for up to seven years in cases.
- Engaging in the sustainable development of the environment.
- Attaining protection of the right to life under Article 21 of the Constitution.
Incorrect
Solution (b)
Statement 1 Statement 2 Statement 3 Statement 4 Statement 5 Correct Incorrect Correct Correct Incorrect Environment Pollution (Prevention and Control) Authority: It is established under the provisions of the Environment (Protection) Act, 1986.
National Tiger Conservation Authority (NTCA): It is a statutory body constituted under provisions of the Wildlife (Protection) Act, 1972.
Central Ground Water Authority: It is established under the provisions of the Environment (Protection) Act, 1986.
National Ganga Council: It is established under the provisions of the Environment (Protection) Act, 1986.
The Central Pollution Control Board (CPCB): It is a statutory organisation constituted under the Water (Prevention and Control of Pollution) Act, 1974.
Note:
Environment (Protection) Act, 1986:
The Environment (Protection) Act was enacted in the year 1986.
It was enacted with the main objective to provide the protection and improvement of the environment and for matters connected therewith.
Aims and Objectives of the Environment Protection Act, 1986:
- Implementing the decisions made at the United Nations Conference on Human Environment held in Stockholm.
- Creation of a government authority to regulate industry that can issue direct orders including closure orders.
- Coordinating activities of different agencies that are operating under the existing laws.
- Enacting regular laws for the protection of the environment.
- Imposing punishments and penalties on those who endanger the environment, safety and health. For each failure or contravention, the punishment includes a prison term of up to five years or a fine of up to Rs. 1 lakh, or both. This can also be extended for up to seven years in cases.
- Engaging in the sustainable development of the environment.
- Attaining protection of the right to life under Article 21 of the Constitution.
-
Question 9 of 35
9. Question
Which among the following rights is/are given to forest dwellers under The Scheduled Tribes and Other Traditional Forest Dwellers (Recognition of Forest Rights) Act, 2006?
- Right to hold and live in the forest land under the individual or common occupation
- Right of ownership, access to collect, use, and dispose of minor forest produce.
- Right of access to biodiversity and community right to intellectual property and traditional knowledge related to biodiversity and cultural diversity.
How many of the above statements are correct?
Correct
Solution (c)
Statement 1 Statement 2 Statement 3 Correct Correct Correct Right to hold and live in the forest land under the individual or common occupation. Right of ownership, access to collect, Use, and dispose of minor forest produce. Right of access to biodiversity and community right to intellectual property and traditional knowledge related to biodiversity and cultural diversity. Note:
The Scheduled Tribes and Other Traditional Forest Dwellers (Recognition of Forest Rights) Act, 2006:
- To address the adverse living conditions of many tribal families living in forests was on account of non-recognition and vesting of pre-existing rights, a landmark legislation viz.
- Scheduled Tribes and Other Traditional Forest Dwellers (Recognition of Forest Rights) Act, 2006, has been enacted to recognize and vest the forest rights and occupation of forest land in forest dwelling Scheduled Tribes and other traditional forest dwellers, who have been residing in such forests for generations, but whose rights could not be recorded.
- This Act not only recognizes the rights to hold and live in the forest land under the individual or common occupation for habitation or for self-cultivation for livelihood, but also grants several other rights to ensure their control over forest resources which, inter-alia, include right of ownership, access to collect, use and dispose of minor forest produce, community rights such as Nistar; habitat rights for primitive tribal groups and pre-agricultural communities; right to protect, regenerate or conserve or manage any community forest resource which they have been traditionally protecting and conserving for sustainable use.
- The Act also provides for diversion of forest land for public utility facilities managed by the Government, such as schools, dispensaries, fair price shops, electricity and telecommunication lines, water tanks, etc. with the recommendation of Gram Sabhas.
- In addition, several schemes have been implemented by the Ministry of Tribal Affairs for the benefit of tribal people, including those in the forest areas such as Mechanism for marketing of Minor Forest Produce (MFP) through Minimum Support Price (MSP) and development of Value Chain for MFP”.
- Funds are released out of Special Central Assistance to Tribal Sub Plan for infrastructure work relating to basic services and facilities viz. approach roads, healthcare, primary education, minor irrigation, rainwater harvesting, drinking water, sanitation, community halls, etc. for development of forest villages.
Incorrect
Solution (c)
Statement 1 Statement 2 Statement 3 Correct Correct Correct Right to hold and live in the forest land under the individual or common occupation. Right of ownership, access to collect, Use, and dispose of minor forest produce. Right of access to biodiversity and community right to intellectual property and traditional knowledge related to biodiversity and cultural diversity. Note:
The Scheduled Tribes and Other Traditional Forest Dwellers (Recognition of Forest Rights) Act, 2006:
- To address the adverse living conditions of many tribal families living in forests was on account of non-recognition and vesting of pre-existing rights, a landmark legislation viz.
- Scheduled Tribes and Other Traditional Forest Dwellers (Recognition of Forest Rights) Act, 2006, has been enacted to recognize and vest the forest rights and occupation of forest land in forest dwelling Scheduled Tribes and other traditional forest dwellers, who have been residing in such forests for generations, but whose rights could not be recorded.
- This Act not only recognizes the rights to hold and live in the forest land under the individual or common occupation for habitation or for self-cultivation for livelihood, but also grants several other rights to ensure their control over forest resources which, inter-alia, include right of ownership, access to collect, use and dispose of minor forest produce, community rights such as Nistar; habitat rights for primitive tribal groups and pre-agricultural communities; right to protect, regenerate or conserve or manage any community forest resource which they have been traditionally protecting and conserving for sustainable use.
- The Act also provides for diversion of forest land for public utility facilities managed by the Government, such as schools, dispensaries, fair price shops, electricity and telecommunication lines, water tanks, etc. with the recommendation of Gram Sabhas.
- In addition, several schemes have been implemented by the Ministry of Tribal Affairs for the benefit of tribal people, including those in the forest areas such as Mechanism for marketing of Minor Forest Produce (MFP) through Minimum Support Price (MSP) and development of Value Chain for MFP”.
- Funds are released out of Special Central Assistance to Tribal Sub Plan for infrastructure work relating to basic services and facilities viz. approach roads, healthcare, primary education, minor irrigation, rainwater harvesting, drinking water, sanitation, community halls, etc. for development of forest villages.
-
Question 10 of 35
10. Question
With respect to Graded Response Action Plan (GRAP), consider the following statements:
- Graded Response Action Plan (GRAP) is a set of emergency measures to deal with air pollution.
- There are five stages in which the Graded Response Action Plan (GRAP) can be activated.
- GRAP is enforced based only on the concentration of PM 2.5 and PM 10.
How many of the above statements are correct?
Correct
Solution (a)
Statement 1 Statement 2 Statement 3 Correct Incorrect Incorrect GRAP is a set of emergency measures that kick in to prevent further deterioration of air quality once it reaches a certain threshold. There are four stages in which the Graded Response Action Plan (GRAP) can be activated. Stage 1 of GRAP is activated when the AQI is in the ‘poor’ category (201 to 300).
The second, third and fourth stages will be activated three days ahead of the AQI reaching the ‘very poor’ category (301 to 400), ‘severe’ category (401 to 450) and ‘severe +’ category (above 450) respectively.
GRAP is being enforced based on the AQI, which takes other pollutants also into account, such as ozone, Sulphur dioxide and oxides of nitrogen, along with PM 2.5 and PM 10. Note:
Graded Response Action Plan (GRAP):
- GRAP is a set of emergency measures that kick in to prevent further deterioration of air quality once it reaches a certain threshold.
- Stage 1 of GRAP is activated when the AQI is in the ‘poor’ category (201 to 300).
- The second, third and fourth stages will be activated three days ahead of the AQI reaching the ‘very poor’ category (301 to 400), ‘severe’ category (401 to 450) and ‘severe +’ category (above 450) respectively.
- For this, the Commission for Air Quality Management (CAQM) is relying on air quality and meteorological forecasts by the Indian Institute of Tropical Meteorology (IITM) and the India Meteorological Department (IMD).
- Measures being imposed under the previous categories will continue even when the subsequent category is activated, that is, if measures under Stage-2 are activated, measures under Stage-1 will continue to remain in place.
- The GRAP was first notified in January 2017 by the Ministry of Environment, Forest and Climate Change.
- This was based on a plan that was submitted by the Central Pollution Control Board (CPCB) in November 2016. According to the notification, the task of implementing the GRAP fell on the now dissolved Environment Pollution (Prevention and Control) Authority for the NCR.
- From 2021 onwards, the GRAP is being implemented by the Commission for Air Quality Management (CAQM).
- The CAQM revised the Graded Response Action Plan from 2022.
How is the GRAP different from 2022 onwards?
- In the version of the GRAP that was notified in 2017, measures kicked in after pollution concentrations reached a certain level.
- This year, measures are pre-emptive and will kick in based on forecasts in an attempt to prevent the AQI from deteriorating further.
- The older version of the GRAP was enforced based only on the concentration of PM2.5 and PM10.
- This year, GRAP is being enforced based on the AQI, which takes other pollutants also into account, such as ozone, Sulphur dioxide and oxides of nitrogen.
Incorrect
Solution (a)
Statement 1 Statement 2 Statement 3 Correct Incorrect Incorrect GRAP is a set of emergency measures that kick in to prevent further deterioration of air quality once it reaches a certain threshold. There are four stages in which the Graded Response Action Plan (GRAP) can be activated. Stage 1 of GRAP is activated when the AQI is in the ‘poor’ category (201 to 300).
The second, third and fourth stages will be activated three days ahead of the AQI reaching the ‘very poor’ category (301 to 400), ‘severe’ category (401 to 450) and ‘severe +’ category (above 450) respectively.
GRAP is being enforced based on the AQI, which takes other pollutants also into account, such as ozone, Sulphur dioxide and oxides of nitrogen, along with PM 2.5 and PM 10. Note:
Graded Response Action Plan (GRAP):
- GRAP is a set of emergency measures that kick in to prevent further deterioration of air quality once it reaches a certain threshold.
- Stage 1 of GRAP is activated when the AQI is in the ‘poor’ category (201 to 300).
- The second, third and fourth stages will be activated three days ahead of the AQI reaching the ‘very poor’ category (301 to 400), ‘severe’ category (401 to 450) and ‘severe +’ category (above 450) respectively.
- For this, the Commission for Air Quality Management (CAQM) is relying on air quality and meteorological forecasts by the Indian Institute of Tropical Meteorology (IITM) and the India Meteorological Department (IMD).
- Measures being imposed under the previous categories will continue even when the subsequent category is activated, that is, if measures under Stage-2 are activated, measures under Stage-1 will continue to remain in place.
- The GRAP was first notified in January 2017 by the Ministry of Environment, Forest and Climate Change.
- This was based on a plan that was submitted by the Central Pollution Control Board (CPCB) in November 2016. According to the notification, the task of implementing the GRAP fell on the now dissolved Environment Pollution (Prevention and Control) Authority for the NCR.
- From 2021 onwards, the GRAP is being implemented by the Commission for Air Quality Management (CAQM).
- The CAQM revised the Graded Response Action Plan from 2022.
How is the GRAP different from 2022 onwards?
- In the version of the GRAP that was notified in 2017, measures kicked in after pollution concentrations reached a certain level.
- This year, measures are pre-emptive and will kick in based on forecasts in an attempt to prevent the AQI from deteriorating further.
- The older version of the GRAP was enforced based only on the concentration of PM2.5 and PM10.
- This year, GRAP is being enforced based on the AQI, which takes other pollutants also into account, such as ozone, Sulphur dioxide and oxides of nitrogen.
-
Question 11 of 35
11. Question
Consider the statements regarding the concept of ‘Panchamrita’ mentioned by India at the 26th Conference of Parties (CoP26) in Glasgow. Which of the following is not a commitment made by India under the ‘Panchamrita’ promises?
Correct
Solution (c)
Option A Option B Option C Option D Correct Correct Incorrect Correct India will reduce its projected carbon emission by one billion tonnes by 2030. India will meet 50 percent of its energy requirements by 2030 with renewable energy. India will get its non-fossil energy capacity to 500 gigawatts by 2030.
India will achieve net-zero by 2070. India will reduce the carbon intensity of its economy by 45 percent by 2030. Note:
Panchamrita Promises at the 26th Conference of Parties (CoP26), Glasgow:
- Prime Minister Narendra Modi proposed a five-fold strategy for India to play its part in helping the world get closer to 1.5 degrees Celsius on the first day of the global climate meet in Glasgow.
- The prime minister euphemistically termed his scheme as ‘Panchamrita’ meaning the ‘Five Ambrosia’.
- ‘Panchamrita’ is a traditional method of mixing five natural foods — milk, ghee, curd, honey and jaggery. These are used in Hindu and Jain worship rituals. It is also used as a technique in Ayurveda.
- Modi’s ‘Panchamrita’ promises include:
- India will get its non-fossil energy capacity to 500 Gigawatt by 2030.
- India will meet 50 per cent of its energy requirements till 2030 with renewable energy.
- India will reduce its projected carbon emission by one billion tonnes by 2030.
- India will reduce the carbon intensity of its economy by 45 per cent by 2030.
- India will achieve net zero by 2070.
Incorrect
Solution (c)
Option A Option B Option C Option D Correct Correct Incorrect Correct India will reduce its projected carbon emission by one billion tonnes by 2030. India will meet 50 percent of its energy requirements by 2030 with renewable energy. India will get its non-fossil energy capacity to 500 gigawatts by 2030.
India will achieve net-zero by 2070. India will reduce the carbon intensity of its economy by 45 percent by 2030. Note:
Panchamrita Promises at the 26th Conference of Parties (CoP26), Glasgow:
- Prime Minister Narendra Modi proposed a five-fold strategy for India to play its part in helping the world get closer to 1.5 degrees Celsius on the first day of the global climate meet in Glasgow.
- The prime minister euphemistically termed his scheme as ‘Panchamrita’ meaning the ‘Five Ambrosia’.
- ‘Panchamrita’ is a traditional method of mixing five natural foods — milk, ghee, curd, honey and jaggery. These are used in Hindu and Jain worship rituals. It is also used as a technique in Ayurveda.
- Modi’s ‘Panchamrita’ promises include:
- India will get its non-fossil energy capacity to 500 Gigawatt by 2030.
- India will meet 50 per cent of its energy requirements till 2030 with renewable energy.
- India will reduce its projected carbon emission by one billion tonnes by 2030.
- India will reduce the carbon intensity of its economy by 45 per cent by 2030.
- India will achieve net zero by 2070.
-
Question 12 of 35
12. Question
Which of the following is not a special plant mentioned in Schedule VI of Wildlife Protection Act,1972?
Correct
Solution (d)
Wildlife Protection Act, 1972:
- Special plants are those plants that are mentioned in Schedule VI of Wildlife Protection Act,1972.
- Cycas Beddomi
- Red Vanda
- Blue Vanda
- Pitcher Plant (Sasuaria lappa)
- Ladies Slipper Orchid
Lantana Camara:
- It is a small perennial shrub, which forms extensive, dense and impenetrable thickets.
- It is native to Central and South America.
- It is an invasive species.
Incorrect
Solution (d)
Wildlife Protection Act, 1972:
- Special plants are those plants that are mentioned in Schedule VI of Wildlife Protection Act,1972.
- Cycas Beddomi
- Red Vanda
- Blue Vanda
- Pitcher Plant (Sasuaria lappa)
- Ladies Slipper Orchid
Lantana Camara:
- It is a small perennial shrub, which forms extensive, dense and impenetrable thickets.
- It is native to Central and South America.
- It is an invasive species.
-
Question 13 of 35
13. Question
Consider the following statements about National Forest Policy, 1988:
- The Government of India declared its first forest policy in 1972.
- The forest policy advocates an end to the system of contractors working in the forests.
Which of the above statements is/are correct?
Correct
Solution (b)
Statement 1 Statement 2 Incorrect Correct The government of India declared its first forest policy in 1952. It recommended that 33% of the total land area of the country should be brought under Forest or tree cover.
The forest policy advocates an end to the system of contractors working in the forests. Note:
National Forest Policy 1952:
- An independent & democratic India saw a lot of new political initiatives.
- Large forest areas of princely states and ‘Zamindaris’ were taken and adoption of the Forest Policy of 1952 which recommended that 33% of the total land area of the country should be brought under Forest or tree cover.
- It provided detailed guidelines for management and protection of forests and wildlife.
National Forest Policy 1988:
This forest policy mainly emphasizes protection, conservation and development of the forests.
Objectives:
- Maintenance of environmental stability through preservation and restoration of ecological balance.
- Conservation of forest as natural heritage.
- Check on soil erosion and denudation in catchment areas of rivers, lakes and reservoirs.
- Check on extension of sand dunes in the desert areas of Rajasthan and along coastal tracts.
- Substantial increase in forest cover through massive afforestation and social forestry schemes.
- Steps to meet requirement of fuel wood, fodder, minor forest produce and soil timber of rural and tribal populations.
- Increase in productivity of forest to meet the national need.
- Putting an end to the system of contractors working in the forests.
- Encouragement of efficient utilization of forest produce and optimum substitution of wood.
- Steps to create awareness between peoples and achieve the objectives with minimizing the pressure on existing forests.
- Involvement of people in forest management under Joint Forest Management.
Incorrect
Solution (b)
Statement 1 Statement 2 Incorrect Correct The government of India declared its first forest policy in 1952. It recommended that 33% of the total land area of the country should be brought under Forest or tree cover.
The forest policy advocates an end to the system of contractors working in the forests. Note:
National Forest Policy 1952:
- An independent & democratic India saw a lot of new political initiatives.
- Large forest areas of princely states and ‘Zamindaris’ were taken and adoption of the Forest Policy of 1952 which recommended that 33% of the total land area of the country should be brought under Forest or tree cover.
- It provided detailed guidelines for management and protection of forests and wildlife.
National Forest Policy 1988:
This forest policy mainly emphasizes protection, conservation and development of the forests.
Objectives:
- Maintenance of environmental stability through preservation and restoration of ecological balance.
- Conservation of forest as natural heritage.
- Check on soil erosion and denudation in catchment areas of rivers, lakes and reservoirs.
- Check on extension of sand dunes in the desert areas of Rajasthan and along coastal tracts.
- Substantial increase in forest cover through massive afforestation and social forestry schemes.
- Steps to meet requirement of fuel wood, fodder, minor forest produce and soil timber of rural and tribal populations.
- Increase in productivity of forest to meet the national need.
- Putting an end to the system of contractors working in the forests.
- Encouragement of efficient utilization of forest produce and optimum substitution of wood.
- Steps to create awareness between peoples and achieve the objectives with minimizing the pressure on existing forests.
- Involvement of people in forest management under Joint Forest Management.
-
Question 14 of 35
14. Question
Montreal Process, Near East Process, Bhopal-India Process are related to:
Correct
Solution (d)
Note:
Montreal Process:
- The Montreal Process, officially known as the Montreal Process Working Group on Criteria and Indicators for the Conservation and Sustainable Management of Temperate and Boreal Forests, is a voluntary agreement on sustainable forest management.
- It was formed in Geneva, Switzerland in June 1994 as a result of the Rio Forest Principles developed at the 1992 Earth Summit.
Near East Process:
- FAO/UNEP Expert Meeting on Criteria and Indicators for Sustainable Forest Management for countries in the region identified 7 criteria and 65 indicators for sustainable forest management at the regional and national levels.
- The guidelines are intended to assist countries in assessing and measuring the sustainability of forest management activities.
Bhopal-India Process:
- It is one of the nine global initiatives on Criteria and Indicators approach for Sustainable Forest Management.
- Its genesis is a Workshop “Regional Initiative for the Development and Implementation of National Level Criteria and Indicators for the Sustainable Management of Dry Forests in Asia” held at IIFM Bhopal, India in November 1999, where nine countries of the Dry Zone Asia (Bangladesh, Bhutan, China, India, Mongolia, Myanmar, Nepal, Sri Lanka, and Thailand) participated.
- Working on 8 Criteria and 49 Indicators of Bhopal-India Process, India developed its National set of 8 Criteria and 37 Indicators for the Sustainable Management of its Natural Forests.
- These 8 Criteria and 37 Indicators have been adopted in the National Working Plan Code 2014, a guiding document for the preparation of Working/ Management Plans in the country at Forest Management Unit level.
Incorrect
Solution (d)
Note:
Montreal Process:
- The Montreal Process, officially known as the Montreal Process Working Group on Criteria and Indicators for the Conservation and Sustainable Management of Temperate and Boreal Forests, is a voluntary agreement on sustainable forest management.
- It was formed in Geneva, Switzerland in June 1994 as a result of the Rio Forest Principles developed at the 1992 Earth Summit.
Near East Process:
- FAO/UNEP Expert Meeting on Criteria and Indicators for Sustainable Forest Management for countries in the region identified 7 criteria and 65 indicators for sustainable forest management at the regional and national levels.
- The guidelines are intended to assist countries in assessing and measuring the sustainability of forest management activities.
Bhopal-India Process:
- It is one of the nine global initiatives on Criteria and Indicators approach for Sustainable Forest Management.
- Its genesis is a Workshop “Regional Initiative for the Development and Implementation of National Level Criteria and Indicators for the Sustainable Management of Dry Forests in Asia” held at IIFM Bhopal, India in November 1999, where nine countries of the Dry Zone Asia (Bangladesh, Bhutan, China, India, Mongolia, Myanmar, Nepal, Sri Lanka, and Thailand) participated.
- Working on 8 Criteria and 49 Indicators of Bhopal-India Process, India developed its National set of 8 Criteria and 37 Indicators for the Sustainable Management of its Natural Forests.
- These 8 Criteria and 37 Indicators have been adopted in the National Working Plan Code 2014, a guiding document for the preparation of Working/ Management Plans in the country at Forest Management Unit level.
-
Question 15 of 35
15. Question
Consider the following statements regarding Global Alliance for Climate-Smart Agriculture (GACSA):
- It is an inclusive, voluntary and action-oriented multi-stakeholder platform on Climate-Smart Agriculture (CSA).
- India is a member state to this alliance.
- Membership in the Alliance creates binding obligations and determines the nature of participation.
How many of the above statements are correct?
Correct
Solution (b)
Statement 1 Statement 2 Statement 3 Correct Correct Incorrect GACSA is an inclusive, voluntary and action-oriented multi-stakeholder platform on Climate-Smart Agriculture (CSA). India is a member state to this alliance. Membership in the Alliance does not create any binding obligations and each member individually determines the nature of its participation. Note:
Global Alliance for Climate-Smart Agriculture:
- GACSA is an inclusive, voluntary and action-oriented multi-stakeholder platform on Climate-Smart Agriculture (CSA).
- Its vision is to improve food security, nutrition and resilience in the face of climate change.
- India is a member state to this alliance.
- Membership in the Alliance does not create any binding obligations and each member individually determines the nature of its participation.
- GACSA aims to catalyze and help create transformational partnerships to encourage actions that reflect an integrated approach to the three pillars of CSA.
- Recognizing the importance of empowering farmers, GACSA facilitates dialogue, knowledge exchange and partnerships through an open, diverse and inclusive multi-stakeholder platform, to catalyze actions on enhancing agriculture, forestry, livestock and fisheries practices and systems that increase productivity in a sustainable way, improve resilience and adaptation and reduce/sequester emissions.
Incorrect
Solution (b)
Statement 1 Statement 2 Statement 3 Correct Correct Incorrect GACSA is an inclusive, voluntary and action-oriented multi-stakeholder platform on Climate-Smart Agriculture (CSA). India is a member state to this alliance. Membership in the Alliance does not create any binding obligations and each member individually determines the nature of its participation. Note:
Global Alliance for Climate-Smart Agriculture:
- GACSA is an inclusive, voluntary and action-oriented multi-stakeholder platform on Climate-Smart Agriculture (CSA).
- Its vision is to improve food security, nutrition and resilience in the face of climate change.
- India is a member state to this alliance.
- Membership in the Alliance does not create any binding obligations and each member individually determines the nature of its participation.
- GACSA aims to catalyze and help create transformational partnerships to encourage actions that reflect an integrated approach to the three pillars of CSA.
- Recognizing the importance of empowering farmers, GACSA facilitates dialogue, knowledge exchange and partnerships through an open, diverse and inclusive multi-stakeholder platform, to catalyze actions on enhancing agriculture, forestry, livestock and fisheries practices and systems that increase productivity in a sustainable way, improve resilience and adaptation and reduce/sequester emissions.
-
Question 16 of 35
16. Question
Consider the following statements about the National Wild Life Action Plan for 2017-2031:
- This is the first time that an action plan on wildlife is recognizing the impact of climate change on wildlife
- This is the first National Wildlife Action Plan (NWAP) released by Union Ministry of Environment, Forests and Climate Change (MoEFCC).
- Plan also suggests private sector participation in the wildlife conservation process.
How many of the above statements are correct?
Correct
Solution (b)
Statement 1 Statement 2 Statement 3 Correct Incorrect Correct This is the first time that an action plan on wildlife is recognizing the impact of climate change on wildlife. This is third National Wildlife Action Plan (NWAP) released by Union Ministry of Environment, Forests and Climate Change (MoEFCC). Plan also suggests private sector participation in the wildlife conservation process. Note:
National Wild Life Action Plan for 2017-2031:
- The first Wildlife Action Plan was released from 1983 to 2001 and the second for 2002 to 2016.
- The Union Ministry of Environment, Forests and Climate Change (MoEFCC) announced the third National Wildlife Action Plan for 2017-2031.
Features:
- This is the first time that an action plan on wildlife is recognizing the impact of climate change on wildlife.
- The plan focuses on integrating climate change mitigation actions into the wildlife management planning process.
- It suggests planting along ecological gradients and assisted wildlife migration because climate change has caused the death of certain flora.
- Special focus on habitat conservation in coastal, marine, and inland aquatic ecosystems and also the recovery of threatened species.
- The plan talks about the issue of animal-human conflict and its impact on wildlife habitats such as its shrinkage, deterioration, and fragmentation.
- Highlighting the importance of people’s participation in this regard, the plan encourages awareness of conservation, ecodevelopment, education, training, and outreach programs for people.
- It also suggests private sector participation in the wildlife conservation process.
Incorrect
Solution (b)
Statement 1 Statement 2 Statement 3 Correct Incorrect Correct This is the first time that an action plan on wildlife is recognizing the impact of climate change on wildlife. This is third National Wildlife Action Plan (NWAP) released by Union Ministry of Environment, Forests and Climate Change (MoEFCC). Plan also suggests private sector participation in the wildlife conservation process. Note:
National Wild Life Action Plan for 2017-2031:
- The first Wildlife Action Plan was released from 1983 to 2001 and the second for 2002 to 2016.
- The Union Ministry of Environment, Forests and Climate Change (MoEFCC) announced the third National Wildlife Action Plan for 2017-2031.
Features:
- This is the first time that an action plan on wildlife is recognizing the impact of climate change on wildlife.
- The plan focuses on integrating climate change mitigation actions into the wildlife management planning process.
- It suggests planting along ecological gradients and assisted wildlife migration because climate change has caused the death of certain flora.
- Special focus on habitat conservation in coastal, marine, and inland aquatic ecosystems and also the recovery of threatened species.
- The plan talks about the issue of animal-human conflict and its impact on wildlife habitats such as its shrinkage, deterioration, and fragmentation.
- Highlighting the importance of people’s participation in this regard, the plan encourages awareness of conservation, ecodevelopment, education, training, and outreach programs for people.
- It also suggests private sector participation in the wildlife conservation process.
-
Question 17 of 35
17. Question
Consider the following statements about Wild Life Protection Act 1972:
- It helped India become a party to the Convention on International Trade in Endangered Species of Wild Fauna and Flora (CITES).
- The National Board for Wildlife was constituted as a statutory organization under the provisions of this Act.
- The Act also provided for the establishment of the National Tiger Conservation Authority.
How many of the above statements are correct?
Correct
Solution (c)
Statement 1 Statement 2 Statement 3 Correct Correct Correct It helped India become a party to the Convention on International Trade in Endangered Species of Wild Fauna and Flora (CITES). The National Board for Wildlife was constituted as a statutory organization under the provisions of this Act. The Act also provided for the establishment of the National Tiger Conservation Authority. Note:
Wild Life Protection Act 1972:
- The Indian Parliament enacted the Wildlife (Protection) Act in 1972, which provides for the safeguard and protection of the wildlife (flora and fauna) in the country.
- This Act provides for the protection of a listed species of animals, birds, and plants, and also for the establishment of a network of ecologically-important protected areas in the country.
- The Act provides for the formation of wildlife advisory boards, wildlife wardens, specifies their powers and duties, etc.
- It helped India become a party to the Convention on International Trade in Endangered Species of Wild Fauna and Flora (CITES).
- CITES is a multilateral treaty with the objective of protecting endangered animals and plants.
- It is also known as the Washington Convention and was adopted as a result of a meeting of IUCN members.
- For the first time, a comprehensive list of the endangered wildlife of the country was prepared.
- The Act prohibited the hunting of endangered species.
- Scheduled animals are prohibited from being traded as per the Act’s provisions.
- The Act provides for licenses for the sale, transfer, and possession of some wildlife species.
- It provides for the establishment of wildlife sanctuaries, national parks, etc.
- Its provisions paved the way for the formation of the Central Zoo Authority. This is the central body responsible for the oversight of zoos in India. It was established in 1992.
- The Act created six schedules which gave varying degrees of protection to classes of flora and fauna.
- Schedule I and Schedule II (Part II) get absolute protection, and offences under these schedules attract the maximum penalties.
- The schedules also include species that may be hunted.
- The National Board for Wildlife was constituted as a statutory organization under the provisions of this Act.
- This is an advisory board that offers advice to the central government on issues of wildlife conservation in India.
- It is also the apex body to review and approve all matters related to wildlife, projects of national parks, sanctuaries, etc.
- The chief function of the Board is to promote the conservation and development of wildlife and forests.
- It is chaired by the Prime Minister.
- The Act also provided for the establishment of the National Tiger Conservation Authority.
- It is a statutory body of the Ministry of Environment, Forest and Climate Change with an overall supervisory and coordination part, performing capacities as given in the Act.
- Its mandate is to strengthen tiger conservation in India.
- It gives statutory authority to Project Tiger which was launched in 1973 and has put the endangered tiger on a guaranteed path of revival by protecting it from extinction.
Incorrect
Solution (c)
Statement 1 Statement 2 Statement 3 Correct Correct Correct It helped India become a party to the Convention on International Trade in Endangered Species of Wild Fauna and Flora (CITES). The National Board for Wildlife was constituted as a statutory organization under the provisions of this Act. The Act also provided for the establishment of the National Tiger Conservation Authority. Note:
Wild Life Protection Act 1972:
- The Indian Parliament enacted the Wildlife (Protection) Act in 1972, which provides for the safeguard and protection of the wildlife (flora and fauna) in the country.
- This Act provides for the protection of a listed species of animals, birds, and plants, and also for the establishment of a network of ecologically-important protected areas in the country.
- The Act provides for the formation of wildlife advisory boards, wildlife wardens, specifies their powers and duties, etc.
- It helped India become a party to the Convention on International Trade in Endangered Species of Wild Fauna and Flora (CITES).
- CITES is a multilateral treaty with the objective of protecting endangered animals and plants.
- It is also known as the Washington Convention and was adopted as a result of a meeting of IUCN members.
- For the first time, a comprehensive list of the endangered wildlife of the country was prepared.
- The Act prohibited the hunting of endangered species.
- Scheduled animals are prohibited from being traded as per the Act’s provisions.
- The Act provides for licenses for the sale, transfer, and possession of some wildlife species.
- It provides for the establishment of wildlife sanctuaries, national parks, etc.
- Its provisions paved the way for the formation of the Central Zoo Authority. This is the central body responsible for the oversight of zoos in India. It was established in 1992.
- The Act created six schedules which gave varying degrees of protection to classes of flora and fauna.
- Schedule I and Schedule II (Part II) get absolute protection, and offences under these schedules attract the maximum penalties.
- The schedules also include species that may be hunted.
- The National Board for Wildlife was constituted as a statutory organization under the provisions of this Act.
- This is an advisory board that offers advice to the central government on issues of wildlife conservation in India.
- It is also the apex body to review and approve all matters related to wildlife, projects of national parks, sanctuaries, etc.
- The chief function of the Board is to promote the conservation and development of wildlife and forests.
- It is chaired by the Prime Minister.
- The Act also provided for the establishment of the National Tiger Conservation Authority.
- It is a statutory body of the Ministry of Environment, Forest and Climate Change with an overall supervisory and coordination part, performing capacities as given in the Act.
- Its mandate is to strengthen tiger conservation in India.
- It gives statutory authority to Project Tiger which was launched in 1973 and has put the endangered tiger on a guaranteed path of revival by protecting it from extinction.
-
Question 18 of 35
18. Question
Which of the given statements is/are correct regarding Environment Protection Act 1986?
- In the wake of the Bhopal Tragedy, the Government of India enacted the Environment Protection Act of 1986.
- The purpose of the Act is to implement the decisions of the United Nations Conference on the Human Environment of 1972.
Which of the statements given above is/are correct?
Correct
Solution (c)
Statement 1 Statement 2 Correct Correct In the wake of the Bhopal Tragedy, the Government of India enacted the Environment Protection Act of 1986. The purpose of the Act is to implement the decisions of the United Nations Conference on the Human Environment of 1972. Note:
Environment Protection Act 1986:
- The Environment (Protection) Act (EPA) was enacted in 1986 with the objective of providing the protection and improvement of the environment.
- In the wake of the Bhopal Tragedy, the Government of India enacted the Environment Protection Act of 1986.
- It empowers the Central Government to establish authorities charged with the mandate of preventing environmental pollution in all its forms and to tackle specific environmental problems that are peculiar to different parts of the country.
- The Act is one of the most comprehensive legislations with a pretext to protection and improvement of the environment.
- The roots of the enactment of the EPA lies in the United Nations Conference on the Human Environment held at Stockholm in June, 1972 (Stockholm Conference), in which India participated, to take appropriate steps for the improvement of the human environment.
- The Act implements the decisions made at the Stockholm Conference.
Provisions:
The EPA empowers the Centre to “take all such measures as it deems necessary” in the domain of environmental protection.
- Under the law, it can coordinate and execute nationwide programmes and plans to further environmental protection.
- It can mandate environmental quality standards, particularly those concerning the emission or discharge of environmental pollutants.
- This law can impose restrictions on the location of industries.
- The law gives the government the power of entry for examination, testing of equipment and other purposes and power to analyse the sample of air, water, soil or any other substance from any place.
- The EPA explicitly bars the discharge of environmental pollutants in excess of prescribed regulatory standards.
- There is also in place a specific provision for handling hazardous substances, which is prohibited unless in compliance with regulatory requirements.
- The Act empowers any person, apart from authorised government officers, to file a complaint in a court regarding any contravention of the provisions of the Act.
Incorrect
Solution (c)
Statement 1 Statement 2 Correct Correct In the wake of the Bhopal Tragedy, the Government of India enacted the Environment Protection Act of 1986. The purpose of the Act is to implement the decisions of the United Nations Conference on the Human Environment of 1972. Note:
Environment Protection Act 1986:
- The Environment (Protection) Act (EPA) was enacted in 1986 with the objective of providing the protection and improvement of the environment.
- In the wake of the Bhopal Tragedy, the Government of India enacted the Environment Protection Act of 1986.
- It empowers the Central Government to establish authorities charged with the mandate of preventing environmental pollution in all its forms and to tackle specific environmental problems that are peculiar to different parts of the country.
- The Act is one of the most comprehensive legislations with a pretext to protection and improvement of the environment.
- The roots of the enactment of the EPA lies in the United Nations Conference on the Human Environment held at Stockholm in June, 1972 (Stockholm Conference), in which India participated, to take appropriate steps for the improvement of the human environment.
- The Act implements the decisions made at the Stockholm Conference.
Provisions:
The EPA empowers the Centre to “take all such measures as it deems necessary” in the domain of environmental protection.
- Under the law, it can coordinate and execute nationwide programmes and plans to further environmental protection.
- It can mandate environmental quality standards, particularly those concerning the emission or discharge of environmental pollutants.
- This law can impose restrictions on the location of industries.
- The law gives the government the power of entry for examination, testing of equipment and other purposes and power to analyse the sample of air, water, soil or any other substance from any place.
- The EPA explicitly bars the discharge of environmental pollutants in excess of prescribed regulatory standards.
- There is also in place a specific provision for handling hazardous substances, which is prohibited unless in compliance with regulatory requirements.
- The Act empowers any person, apart from authorised government officers, to file a complaint in a court regarding any contravention of the provisions of the Act.
-
Question 19 of 35
19. Question
Consider the following statements regarding Plastic Waste Management Rules, 2016:
- The Rules aim at promoting use of plastic waste for road construction.
- The Rules do not mention plastic material for packaging gutkha, pan masala and tobacco.
Which of the above statements is/are correct?
Correct
Solution (c)
Statement 1 Statement 2 Correct Correct The Rules aim at promoting use of plastic waste for road construction. The Rules do not mention plastic material for packaging gutkha, pan masala and tobacco. Note:
Plastic Waste Management Rules, 2016:
- The new plastic waste management rules is aimed at reducing 6,000 tonnes of uncollected plastic waste generated daily by targeting manufacturers and industries by using a new principle called the Extended Producers’ Responsibility (EPR) Act.
- The other modalities of the new plastic management rules are as follows:
- Increase minimum thickness of plastic carry bags from 40 to 50 microns and stipulate minimum thickness of 50 micron for plastic sheets also to facilitate collection and recycle of plastic waste.
- Expand the jurisdiction of applicability from the municipal area to rural areas, because plastic has reached rural areas also.
- To bring in the responsibilities of producers and generators, both in plastic waste management system and to introduce collect back system of plastic waste by the producers/brand owners, as per Extended Producers Responsibility.
- To introduce collection of plastic waste management fee through pre-registration of the producers, importers of plastic carry bags/multilayered packaging and vendors selling the same for establishing the waste management system.
- To promote use of plastic waste for road construction as per Indian Road Congress guidelines or energy recovery, or waste to oil etc. for gainful utilization of waste and also addresses the waste disposal issue.
- To entrust more responsibility on waste generators, namely payment of user charge as prescribed by local authority, collection and handing over of waste by the institutional generator, event organizers.
- To implement these rules more effectively and to give thrust on plastic waste minimization, source segregation, recycling, involving waste pickers, recyclers and waste processors in collection of plastic waste and adopt polluter pays principle for the sustainability of the waste management system.
- Recently, the Ministry of Environment, Forest, and Climate Change announced the Plastic Waste Management (Amendment) Rules, 2022, which notified the instructions on Extended Producer Responsibility (EPR) for plastic packaging.
- Plastic Waste Management Rules 2016 has been amended to fast-track the elimination of single-use plastics and promote alternatives.
- The term Extended Procedure Responsibility means the responsibility of a producer for the environmentally sound management of the product until the end of its life.
Incorrect
Solution (c)
Statement 1 Statement 2 Correct Correct The Rules aim at promoting use of plastic waste for road construction. The Rules do not mention plastic material for packaging gutkha, pan masala and tobacco. Note:
Plastic Waste Management Rules, 2016:
- The new plastic waste management rules is aimed at reducing 6,000 tonnes of uncollected plastic waste generated daily by targeting manufacturers and industries by using a new principle called the Extended Producers’ Responsibility (EPR) Act.
- The other modalities of the new plastic management rules are as follows:
- Increase minimum thickness of plastic carry bags from 40 to 50 microns and stipulate minimum thickness of 50 micron for plastic sheets also to facilitate collection and recycle of plastic waste.
- Expand the jurisdiction of applicability from the municipal area to rural areas, because plastic has reached rural areas also.
- To bring in the responsibilities of producers and generators, both in plastic waste management system and to introduce collect back system of plastic waste by the producers/brand owners, as per Extended Producers Responsibility.
- To introduce collection of plastic waste management fee through pre-registration of the producers, importers of plastic carry bags/multilayered packaging and vendors selling the same for establishing the waste management system.
- To promote use of plastic waste for road construction as per Indian Road Congress guidelines or energy recovery, or waste to oil etc. for gainful utilization of waste and also addresses the waste disposal issue.
- To entrust more responsibility on waste generators, namely payment of user charge as prescribed by local authority, collection and handing over of waste by the institutional generator, event organizers.
- To implement these rules more effectively and to give thrust on plastic waste minimization, source segregation, recycling, involving waste pickers, recyclers and waste processors in collection of plastic waste and adopt polluter pays principle for the sustainability of the waste management system.
- Recently, the Ministry of Environment, Forest, and Climate Change announced the Plastic Waste Management (Amendment) Rules, 2022, which notified the instructions on Extended Producer Responsibility (EPR) for plastic packaging.
- Plastic Waste Management Rules 2016 has been amended to fast-track the elimination of single-use plastics and promote alternatives.
- The term Extended Procedure Responsibility means the responsibility of a producer for the environmentally sound management of the product until the end of its life.
-
Question 20 of 35
20. Question
Consider the following statements regarding the declaration of certain wild animals as ‘Vermin’ in India:
- The State Government has the authority to declare a wild animal as vermin.
- Animals listed in Schedule I of the Wildlife Protection Act, 1972 cannot be declared as vermin.
Which of the statements given above is/are correct?
Correct
Solution (b)
Statement 1 Statement 2 Incorrect Correct As per Section 62 of the Wildlife Protection Act, 1972, States can send a list of wild animals to the Centre requesting it to declare them vermin for selective slaughter. The Central Government may by notification, declare any wild animal other than those specified in Schedule I and part II of Schedule II of the law to be vermin for any area for a given period of time. Note:
Vermin:
- Vermin means wild animals which are believed to be harmful.
- Legally speaking, if a species is declared vermin, that particular species can be hunted or culled without restriction.
- Any species can be declared vermin except:
- Species which are listed in Schedule I of Wildlife Protection Act 1972
- Species which are listed in Part II of Schedule II of WPA 1972
- Section 62 of the Wildlife Protection Act, 1972 empowers the states to send a list of wild animals to the Centre requesting it to declare them vermin for selective slaughter.
- For the period the notification is in force such wild animal shall be included in Schedule V of the law, depriving them of any protection under that law.
- The hunted wildlife is declared as government property and it imposes restrictions on how these carcasses must be disposed of.
Incorrect
Solution (b)
Statement 1 Statement 2 Incorrect Correct As per Section 62 of the Wildlife Protection Act, 1972, States can send a list of wild animals to the Centre requesting it to declare them vermin for selective slaughter. The Central Government may by notification, declare any wild animal other than those specified in Schedule I and part II of Schedule II of the law to be vermin for any area for a given period of time. Note:
Vermin:
- Vermin means wild animals which are believed to be harmful.
- Legally speaking, if a species is declared vermin, that particular species can be hunted or culled without restriction.
- Any species can be declared vermin except:
- Species which are listed in Schedule I of Wildlife Protection Act 1972
- Species which are listed in Part II of Schedule II of WPA 1972
- Section 62 of the Wildlife Protection Act, 1972 empowers the states to send a list of wild animals to the Centre requesting it to declare them vermin for selective slaughter.
- For the period the notification is in force such wild animal shall be included in Schedule V of the law, depriving them of any protection under that law.
- The hunted wildlife is declared as government property and it imposes restrictions on how these carcasses must be disposed of.
-
Question 21 of 35
21. Question
Gudavi Bird Sanctuary is engulfed with natural beauty and rich biodiversity, attracting tourists from different parts of the world. It is covered by moist deciduous forest species interspersed with grassy patches. It is engulfed with natural beauty and rich biodiversity, attracting tourists from different parts of the world.
The above bird sanctuary is located in which of the following states of India?
Correct
Solution (d)
Gudavi Bird Sanctuary is engulfed with natural beauty and rich biodiversity, attracting tourists from different parts of the world. It is covered by moist deciduous forest species interspersed with grassy patches. It is engulfed with natural beauty and rich biodiversity, attracting tourists from different parts of the world. It is located in Karnataka. Hence option d is correct.
Incorrect
Solution (d)
Gudavi Bird Sanctuary is engulfed with natural beauty and rich biodiversity, attracting tourists from different parts of the world. It is covered by moist deciduous forest species interspersed with grassy patches. It is engulfed with natural beauty and rich biodiversity, attracting tourists from different parts of the world. It is located in Karnataka. Hence option d is correct.
-
Question 22 of 35
22. Question
Consider the following statements regarding Election Commission of India:
- Originally the commission had only one election commissioner but was made a multi-member body through the 42nd Amendment Act.
- The Chief Election Commissioner can be removed from office only by the order of the President, just like a judge of the Supreme Court.
Which of the statements given above is/are correct?
Correct
Solution (b)
Election Commission of India
- Originally the commission had only one election commissioner but after the Election Commissioner Amendment Act 1989, it has been made a multi-member body. Hence statement 1 is incorrect.
- The Chief Election Commissioner can be removed from office only by the order of the President, just like a judge of the Supreme Court. Hence statement 2 is correct.
Incorrect
Solution (b)
Election Commission of India
- Originally the commission had only one election commissioner but after the Election Commissioner Amendment Act 1989, it has been made a multi-member body. Hence statement 1 is incorrect.
- The Chief Election Commissioner can be removed from office only by the order of the President, just like a judge of the Supreme Court. Hence statement 2 is correct.
-
Question 23 of 35
23. Question
Consider the following statements about the Banni Festival:
- It is a traditional stick-fight celebrated on the night of Dussehra celebration every year.
- It was celebrated by people under the Vijayanagara Empire.
Which of the statements given above is/are correct?
Correct
Solution (c)
- Banni Festival is a traditional stick-fight celebrated on the night of Dussehra celebration every year. Hence statement 1 is correct.
- It is celebrated in the state of Andhra Pradesh.
- The objective of this event was to snatch the idols from God’s team, leading to a fierce battle known as the Banni Fight.
- It is celebrated on account of the victory of Lord Mala Malleswara Swamy and Goddess Parvati over demonish Mani and Mallasura, who troubled the people in the Devaragattu region.
- It was celebrated by people under the Vijayanagara Empire. Hence statement 2 is correct.
Incorrect
Solution (c)
- Banni Festival is a traditional stick-fight celebrated on the night of Dussehra celebration every year. Hence statement 1 is correct.
- It is celebrated in the state of Andhra Pradesh.
- The objective of this event was to snatch the idols from God’s team, leading to a fierce battle known as the Banni Fight.
- It is celebrated on account of the victory of Lord Mala Malleswara Swamy and Goddess Parvati over demonish Mani and Mallasura, who troubled the people in the Devaragattu region.
- It was celebrated by people under the Vijayanagara Empire. Hence statement 2 is correct.
-
Question 24 of 35
24. Question
Consider the following statements about the Operation Chakra II:
- It aims to fight against transnationally organized cyber-enabled financial crimes in India.
- It was launched by the global money laundering and terrorist financing watchdog Financial Action Task Force (FATF).
Which of the given statement/s is/are correct?
Correct
Solution (a)
- Operation Chakra II aims to fight against transnationally organized cyber-enabled financial crimes in India. Hence statement 1 is correct.
- It aims to tackle transnational organized cyber-enabled financial crimes within India. These crimes often involve fraudulent activities conducted online.
- The CBI has partnered with technology giants Microsoft and Amazon, as well as with national and international law enforcement agencies.
- It was launched by the Central Bureau of Investigation (CBI). Hence statement 2 is incorrect.
Incorrect
Solution (a)
- Operation Chakra II aims to fight against transnationally organized cyber-enabled financial crimes in India. Hence statement 1 is correct.
- It aims to tackle transnational organized cyber-enabled financial crimes within India. These crimes often involve fraudulent activities conducted online.
- The CBI has partnered with technology giants Microsoft and Amazon, as well as with national and international law enforcement agencies.
- It was launched by the Central Bureau of Investigation (CBI). Hence statement 2 is incorrect.
-
Question 25 of 35
25. Question
Consider the following statements about ‘Infantile Hypophosphatasia’:
- It is a rare genetic disease in which the patient’s bones and teeth demineralise, making it fragile and prone to fractures.
- It has noticeable abnormalities at birth, but complications become apparent within the first six years of life.
- Asfotase alfa is approved for enzyme replacement therapy (ERT) in patients with pediatric-onset hypophosphatasia.
How many of the above statements are correct?
Correct
Solution (b)
- Infantile Hypophosphatasia is a rare genetic disease in which the patient’s bones and teeth demineralise, making it fragile and prone to fractures. Hence statement 1 is correct.
- It is caused by mutations in the ALPL gene. This is the only gene that causes HPP.
- Genes provide instructions for making proteins that have an important function in the body.
- It may have no noticeable abnormalities at birth, but complications become apparent within the first six months of life. Hence statement 2 is incorrect.
- Sometimes the skull bones fuse, called craniosynostosis, which can lead to a deformed head.
- Affected infants have softened, weakened, and deformed bones consistent with rickets.
- Asfotase alfa is approved for enzyme replacement therapy (ERT) in patients with pediatric-onset hypophosphatasia. Hence statement 3 is correct.
- It is associated with healing of the skeletal manifestations of hypophosphatasia as well as improved respiratory and motor function.
Incorrect
Solution (b)
- Infantile Hypophosphatasia is a rare genetic disease in which the patient’s bones and teeth demineralise, making it fragile and prone to fractures. Hence statement 1 is correct.
- It is caused by mutations in the ALPL gene. This is the only gene that causes HPP.
- Genes provide instructions for making proteins that have an important function in the body.
- It may have no noticeable abnormalities at birth, but complications become apparent within the first six months of life. Hence statement 2 is incorrect.
- Sometimes the skull bones fuse, called craniosynostosis, which can lead to a deformed head.
- Affected infants have softened, weakened, and deformed bones consistent with rickets.
- Asfotase alfa is approved for enzyme replacement therapy (ERT) in patients with pediatric-onset hypophosphatasia. Hence statement 3 is correct.
- It is associated with healing of the skeletal manifestations of hypophosphatasia as well as improved respiratory and motor function.
-
Question 26 of 35
26. Question
Consider the following statements about the Roman Space Telescope (RST):
- It is an Indian Space Research Organisation (ISRO)’s deep space infrared observatory.
- It answers essential astrophysics questions about dark energy, dark matter, exoplanets, and infrared astrophysics.
Which of the statements given above is/are correct?
Correct
Solution (b)
- The Roman Space Telescope (RST) is a NASA’s deep space infrared observatory. Hence statement 1 is incorrect.
- It is also known as the Nancy Grace Roman Space Telescope.
- Though originally referred to as the Wide Field Infrared Space Telescope (WFIRST), NASA renamed the mission in 2020 after Nancy Grace Roman, NASA’s first chief astronomer.
- It answers essential astrophysics questions about dark energy, dark matter, exoplanets, and infrared astrophysics. Hence statement 2 is correct.
- It is set to launch around 2026 or 2027 and is projected to last for five years.
Incorrect
Solution (b)
- The Roman Space Telescope (RST) is a NASA’s deep space infrared observatory. Hence statement 1 is incorrect.
- It is also known as the Nancy Grace Roman Space Telescope.
- Though originally referred to as the Wide Field Infrared Space Telescope (WFIRST), NASA renamed the mission in 2020 after Nancy Grace Roman, NASA’s first chief astronomer.
- It answers essential astrophysics questions about dark energy, dark matter, exoplanets, and infrared astrophysics. Hence statement 2 is correct.
- It is set to launch around 2026 or 2027 and is projected to last for five years.
-
Question 27 of 35
27. Question
Consider the following statements about Aerogels:
- They are composed mostly of air and can be used to remove contaminants.
- They are solid, rigid, and dry materials that do not resemble a gel in their physical properties.
Which of the given statement/s is/are correct?
Correct
Solution (c)
- Aerogels are composed mostly of air and can be used to remove contaminants. Hence statement 1 is correct.
- They are also known as ‘solid air’ or ‘frozen smoke’, are excellent adsorbents (a solid substance used to remove contaminants), and are incredibly lightweight solids composed mostly of air.
- In addition, they offer advantages like adjustable surface chemistry, low density, and a highly porous structure.
- They are solid, rigid, and dry materials that do not resemble a gel in their physical properties: the name comes from the fact that they are made from gels. Hence statement 2 is correct.
Incorrect
Solution (c)
- Aerogels are composed mostly of air and can be used to remove contaminants. Hence statement 1 is correct.
- They are also known as ‘solid air’ or ‘frozen smoke’, are excellent adsorbents (a solid substance used to remove contaminants), and are incredibly lightweight solids composed mostly of air.
- In addition, they offer advantages like adjustable surface chemistry, low density, and a highly porous structure.
- They are solid, rigid, and dry materials that do not resemble a gel in their physical properties: the name comes from the fact that they are made from gels. Hence statement 2 is correct.
-
Question 28 of 35
28. Question
Consider the following statements about Curcuma kakchingense:
- It belongs to the gymnospermic family Zingiberaceae which includes Curcuma, Gingers, and Cardamom.
- It is used in cuisines, traditional medicines, spices, dyes, perfumes, cosmetics, and as ornamental plants.
Which of the given statement/s is/are correct?
Correct
Solution (b)
Context: Researchers in Manipur have discovered a new flowering plant species in Kakching district. They have named the new plant species Curcuma kakchingense.
- Curcuma kakchingense belongs to the angiospermic family Zingiberaceae which includes Curcuma, Gingers, and Cardamom. Hence statement 1 is incorrect.
- The new flowering plant species is a robust plant, as tall as eight feet, having large terminal inflorescence.
- The species has a close resemblance to Curcuma phrayawan, a species reported from Thailand.
- Its conservation status remains Data Deficient under the International Union for Conservation of Nature (IUCN) Red List category.
- It is used in cuisines, traditional medicines, spices, dyes, perfumes, cosmetics, and as ornamental plants. Hence statement 2 is correct.
Incorrect
Solution (b)
Context: Researchers in Manipur have discovered a new flowering plant species in Kakching district. They have named the new plant species Curcuma kakchingense.
- Curcuma kakchingense belongs to the angiospermic family Zingiberaceae which includes Curcuma, Gingers, and Cardamom. Hence statement 1 is incorrect.
- The new flowering plant species is a robust plant, as tall as eight feet, having large terminal inflorescence.
- The species has a close resemblance to Curcuma phrayawan, a species reported from Thailand.
- Its conservation status remains Data Deficient under the International Union for Conservation of Nature (IUCN) Red List category.
- It is used in cuisines, traditional medicines, spices, dyes, perfumes, cosmetics, and as ornamental plants. Hence statement 2 is correct.
-
Question 29 of 35
29. Question
With reference to the Central Water Commission (CWC), consider the following statements:
- It works as an attached office of the Ministry of Jal Shakti.
- It handles issues related to both surface water and groundwater in India.
Which of the statements given above is/are correct?
Correct
Solution (a)
- Central Water Commission (CWC) is the technical organization of India. It is an attached office of the Department of Water Resources, River Development and Ganga Rejuvenation, Ministry of Jal Shakti, Government of India. Hence statement 1 is correct.
- The Commission is charged with initiating, coordinating, and advancing plans for the control, conservation, and utilisation of water resources across the nation for flood control, irrigation, navigation, drinking water supply, and hydropower development in consultation with the State Governments concerned. It carries out any necessary research, development, and execution of such plans.
- CWC deals with only surface water. Central Groundwater Board (CGWB) handles groundwater. Hence statement 2 is incorrect.
Incorrect
Solution (a)
- Central Water Commission (CWC) is the technical organization of India. It is an attached office of the Department of Water Resources, River Development and Ganga Rejuvenation, Ministry of Jal Shakti, Government of India. Hence statement 1 is correct.
- The Commission is charged with initiating, coordinating, and advancing plans for the control, conservation, and utilisation of water resources across the nation for flood control, irrigation, navigation, drinking water supply, and hydropower development in consultation with the State Governments concerned. It carries out any necessary research, development, and execution of such plans.
- CWC deals with only surface water. Central Groundwater Board (CGWB) handles groundwater. Hence statement 2 is incorrect.
-
Question 30 of 35
30. Question
Consider the following statements about the National Investment and Infrastructure Fund (NIIF):
- It is India’s first sovereign wealth fund, acting as an Alternative Investment Fund under SEBI regulations.
- Under NIIF, the Master Fund invests largely in operating assets in core infrastructure sectors such as highways, ports, airports, electricity, etc.
Which of the given statement/s is/are correct?
Correct
Solution (c)
- The National Investment and Infrastructure Fund (NIIF) is India’s first sovereign wealth fund, acting as an Alternative Investment Fund under SEBI regulations. Hence statement 1 is correct.
- It is a platform for national and international investors to invest equity capital in domestic economic and social infrastructure.
- It manages three types of Funds: Master Funds, Fund of Funds, and Strategic Investment Funds.
- Under NIIF, the Master Fund invests largely in operating assets in core infrastructure sectors such as highways, ports, airports, electricity, etc. Hence statement 2 is correct.
- Fund of Funds invests in infrastructure and related funds managed by fund managers with an excellent track record. Its main emphasis is on Green Infrastructure, Mid-Income and Affordable Housing, Infrastructure Services, and Allied Sectors.
- Strategic Investment Fund aims at growth and development-stage investments in large-scale projects in various economic and commercial sectors projected to profit from India’s medium- to long-term growth trajectory.
Incorrect
Solution (c)
- The National Investment and Infrastructure Fund (NIIF) is India’s first sovereign wealth fund, acting as an Alternative Investment Fund under SEBI regulations. Hence statement 1 is correct.
- It is a platform for national and international investors to invest equity capital in domestic economic and social infrastructure.
- It manages three types of Funds: Master Funds, Fund of Funds, and Strategic Investment Funds.
- Under NIIF, the Master Fund invests largely in operating assets in core infrastructure sectors such as highways, ports, airports, electricity, etc. Hence statement 2 is correct.
- Fund of Funds invests in infrastructure and related funds managed by fund managers with an excellent track record. Its main emphasis is on Green Infrastructure, Mid-Income and Affordable Housing, Infrastructure Services, and Allied Sectors.
- Strategic Investment Fund aims at growth and development-stage investments in large-scale projects in various economic and commercial sectors projected to profit from India’s medium- to long-term growth trajectory.
-
Question 31 of 35
31. Question
The number of oranges in three baskets are in the ratio 3:4:5. In which ratio the no. of oranges in first two baskets must be increased so that the new ratio becomes 5:4:3?
Correct
Solution (d)
Let, B1 : B2 : B3 = 3x : 4x : 5x and
B1 : B2 : B3 = 5y : 4y : 3y
Number of oranges remain constant in third basket as increase in oranges takes place only in first two baskets.
Hence, 5x = 3y
x = (3y/5) and,
∴ 3x : 4x : 5x (putting the value of x)
9y/5:12y/5:15y/5
= 9y : 12y : 15y
And,
5y : 4y : 3y (multiple by 5) → 25y : 20y : 15y
∴Increment in first basket = 16
And, Increment in second basket = 8
Thus, Required ratio = (16/8)
= 2:1
Incorrect
Solution (d)
Let, B1 : B2 : B3 = 3x : 4x : 5x and
B1 : B2 : B3 = 5y : 4y : 3y
Number of oranges remain constant in third basket as increase in oranges takes place only in first two baskets.
Hence, 5x = 3y
x = (3y/5) and,
∴ 3x : 4x : 5x (putting the value of x)
9y/5:12y/5:15y/5
= 9y : 12y : 15y
And,
5y : 4y : 3y (multiple by 5) → 25y : 20y : 15y
∴Increment in first basket = 16
And, Increment in second basket = 8
Thus, Required ratio = (16/8)
= 2:1
-
Question 32 of 35
32. Question
Find the total number of distinct vehicle numbers that can be formed using two letters followed by two numbers. Letters need to be distinct.
Correct
Solution (b)
Out of 26 alphabets two distinct letters can be chosen in 26P2 ways.
Coming to numbers part, there are 10 ways (any number from 0 to 9 can be chosen) to choose the first digit and similarly another 10 ways to choose the second digit.
Hence, there are totally 10 × 10 = 100 ways.
Combined with letters there are,
26P2 × 100 = 65000 ways to choose vehicle numbers.
Incorrect
Solution (b)
Out of 26 alphabets two distinct letters can be chosen in 26P2 ways.
Coming to numbers part, there are 10 ways (any number from 0 to 9 can be chosen) to choose the first digit and similarly another 10 ways to choose the second digit.
Hence, there are totally 10 × 10 = 100 ways.
Combined with letters there are,
26P2 × 100 = 65000 ways to choose vehicle numbers.
-
Question 33 of 35
33. Question
Free notebooks were distributed equally among children of a class. The number of notebooks each child got was one-eighth of the number of children. Had the number of children been half, each child would have got 16 notebooks. Total how many notebooks were distributed?
Correct
Solution (c)
Let number of children=n
Then, number of books each child will get = n/8
Total books distributed = n × (n/8) = n^2/8
If the number children = n/2, number of books each child will get=16
Total books distributed = (n/2)*16 = 8n
Therefore, (n^2/8) = 8n
n/8 = 8
n = 64
Total number of books distributed
= 8n = 8 * 64 = 512
Incorrect
Solution (c)
Let number of children=n
Then, number of books each child will get = n/8
Total books distributed = n × (n/8) = n^2/8
If the number children = n/2, number of books each child will get=16
Total books distributed = (n/2)*16 = 8n
Therefore, (n^2/8) = 8n
n/8 = 8
n = 64
Total number of books distributed
= 8n = 8 * 64 = 512
-
Question 34 of 35
34. Question
A class photograph has to be taken. The front row consists of 6 girls who are sitting. 20 boys are standing behind. The two corner positions are reserved for the 2 tallest boys. In how many ways can the students be arranged?
Correct
Solution (b)
Two tallest boys can be arranged in 2! ways, rest 18 can be arranged in 18! ways.
Girls can be arranged in 6! ways.
Total number of ways in which all the students can be arranged,
= 2! × 18! × 6!
= 18! × 1440
Note:
N! = N × (N – 1) × (N – 2) × . . . . × 1
So,
18! = 18 × 17 × 16 × 15 . . . . . . . . × 1
Incorrect
Solution (b)
Two tallest boys can be arranged in 2! ways, rest 18 can be arranged in 18! ways.
Girls can be arranged in 6! ways.
Total number of ways in which all the students can be arranged,
= 2! × 18! × 6!
= 18! × 1440
Note:
N! = N × (N – 1) × (N – 2) × . . . . × 1
So,
18! = 18 × 17 × 16 × 15 . . . . . . . . × 1
-
Question 35 of 35
35. Question
Eight people are planning to share equally the cost of a rental car. If one person withdraws from the arrangement and the others share equally the entire cost of the car, then the share of each of the remaining persons increased by
Correct
Solution (a)
Let the cost of the car is x
If 8 people are sharing equally then share of one person will be x/8 .
If one person has to withdraw, so seven people will share equally.
New share will be x/7
Increase in share = (x/7 – x/8)/(x/8) = 1/(7×8)/ (1/8) = 1/7
Thus,
Required number of pebbles,
= 1440 – 22
= 1418
Incorrect
Solution (a)
Let the cost of the car is x
If 8 people are sharing equally then share of one person will be x/8 .
If one person has to withdraw, so seven people will share equally.
New share will be x/7
Increase in share = (x/7 – x/8)/(x/8) = 1/(7×8)/ (1/8) = 1/7
Thus,
Required number of pebbles,
= 1440 – 22
= 1418
All the Best
IASbaba